Đến nội dung

nguyentrungphuc26041999 nội dung

Có 382 mục bởi nguyentrungphuc26041999 (Tìm giới hạn từ 28-04-2020)



Sắp theo                Sắp xếp  

#467139 Tìm tổng các chữ số của $A^{2}$ nếu A = 999......998 (200...

Đã gửi bởi nguyentrungphuc26041999 on 27-11-2013 - 19:59 trong Số học

Tìm tổng các chữ số của $A^{2}$ nếu A = 999......998 (2007 chữ số 9)

ta chứng minh $99..9600..04=99..98^{2}$(n chữ số 9 và số 0)

$99..9600..04=99..9.10^{n+2}+6.10^{n+1}+4$

$\left ( 10^{n}-1\right).10^{n+2}+6.10^{n+1}+4=10^{2n+2}-4.10^{n+1}+4=\left ( 10^{n+1}-2 \right )^{2}=99..98^{2}$(n chữ số 9)

tổng các số chữ số của A là

$9.2007+6+4=18073$




#449727 Tìm số tự nhiên n để a=$n(n^{2}+1)(n^{2}+4)$ ch...

Đã gửi bởi nguyentrungphuc26041999 on 12-09-2013 - 21:52 trong Số học

bài 1: Cho k,m,l là các số tự nhiên đôi một ko có cùng số dư trong phép chia cho 5.CMR trong 3 số A=3k+l+m,B=3l+k+m,C=2k+2l+m có một và chỉ một số chia hết cho 5

bài 2:Tìm số tự nhiên n để a=$n(n^{2}+1)(n^{2}+4)$ chia hết cho 120

baif 3: CMR tồn tại một số tự nhiên gồm toàn chữ số 6 chia hết cho 2003

 

bài 3:(hỏi) đề là 1 và chỉ một phải ko???nếu cm tồn tại 1 số thì chỉ cần chỉ số đó là mấy à???????

bài2 luôn

rõ ràng $n\left ( n^{2}+1 \right )\left ( n^{2}+4 \right )\vdots 5$

vì số chính phương chia 5 dư 1,0,-1

do $n\left ( n^{2}+1 \right )\left ( n^{2}+4 \right )\vdots 3$

$\Rightarrow n\vdots 3$

vì  $n^{2}+1$ không chia hết cho 3

$n^{2}+4$ không chia hết cho 3

bây giờ $n\left ( n^{2}+1 \right )\left ( n^{2}+4 \right )\vdots 8$

do $n$ và $n^{2}+1$ khác tính chẵn lẻ 

$n^{2}+1$ và $n^{2}+4$ khác tính chẵn lẻ, $n^{2}+1$ không chia hết cho 8(số chính phương không chia 8 dư 7)

$\Rightarrow n\left ( n^{2}+4 \right )\vdots 8$

$n$ và $n^{2}+4$ cùng chẵn nên $n$ chẵn 

nếu $n= 4k+2$thì $n\left ( n^{2}+4 \right )\equiv 4\left ( mod 8 \right )$(loại)

$\Rightarrow n\vdots 4$

vậy $n= 12k$ thì $n\left ( n^{2}+1 \right )\left ( n^{2}+4 \right )\vdots 120$




#450066 tìm số tự nhiên n để $3^{n}+1$ là số chính phương

Đã gửi bởi nguyentrungphuc26041999 on 13-09-2013 - 22:17 trong Số học

tìm số tự nhiên n để $3^{n}+1$ là số chính phương

đặt $3^{n}+1= k^{2}$

$\Rightarrow 3^{n}=\left ( k-1 \right )\left ( k+1 \right )$

do 3 nguyên tố 

đặt $k+1= 3^{a}$

$k-1= 3^{b}$

suy ra$a> b$

$\Rightarrow \frac{k+1}{k-1}= 3^{a-b}$

$3^{a-b}$ nguyên

$\Rightarrow k+1\vdots k-1$

$\Rightarrow k=2$ hoặc $\Rightarrow k=3$

đến đây thì dễ rồi

k=2 thì n=1,k=3 thì thôi




#498222 Trận 9 - Bất đẳng thức

Đã gửi bởi nguyentrungphuc26041999 on 10-05-2014 - 15:03 trong Thi giải toán Marathon cấp THCS 2014

Cho $x, y, z$ là các số dương thỏa mãn điều kiện $xyz = 1$. Tìm giá trị nhỏ nhất của biểu thức:  

$$E=\frac{1}{x^3(y+z)}+\frac{1}{y^3(z+x)}+\frac{1}{z^3(x+y)}.$$

Toán thủ ra đề: angleofdarkness

Áp dụng bất đẳng thức Bunhiacopxki dạng phân thức,ta có

$E=\sum \frac{\frac{1}{x^{2}}}{xy+xz}\geq \frac{\left ( \frac{1}{x} +\frac{1}{y}+\frac{1}{z}\right )^{2}}{2\left ( xy+yz+zx \right )}=\frac{xy+yz+zx}{2}$

áp dụng bất đẳng thức Cô-si 

$\frac{xy+yz+xz}{2}\geq \frac{3\sqrt[3]{x^{2}y^{2}z^{2}}}{2}=\frac{3}{2}$

Vậy giá trị nhỏ nhất của $E=\frac{3}{2}$ đạt được khi $a=b=c=1$

 

$d = 10$

$S = 41$




#492246 Trận 7 - PT, HPT đại số

Đã gửi bởi nguyentrungphuc26041999 on 11-04-2014 - 20:42 trong Thi giải toán Marathon cấp THCS 2014

Giải phương trình: $2x^{2}+5x-1=7\sqrt{x^{3}-1}$

Đề thi của l4lzTeoz

Ta có 

$2x^{2}+5x-1=7\sqrt{x^{3}-1}$ ( điều kiện xác định $x\geq 1$)

$\Leftrightarrow 2x^{2}+5x-1=7\sqrt{\left ( x-1 \right )\left ( x^{2}+x+1 \right )}$

đặt $\left\{\begin{matrix} \sqrt{x^{2}+x+1}=a & \\ \sqrt{x-1}=b & \end{matrix}\right.$

$\Rightarrow 2a^{2}+3b^{2}=2x^{2}+5x-1$

$\Leftrightarrow 2a^{2}+3b^{2}=7ab$

$\Leftrightarrow 2a^{2}-7ab+b^{2}=0$

$\left ( 2a-b \right )\left ( a-3b \right )=0$

$\Leftrightarrow \left\{\begin{matrix} 2a=b & \\ a=3b & \end{matrix}\right.$

nếu $2a=b$

$\Leftrightarrow 2\sqrt{x^{2}+x+1}=\sqrt{x-1}$

$\Leftrightarrow 4x^{2}+4x+4=x-1$

$\Leftrightarrow 4x^{2}+3x+5=0$ (vô nghiệm do $x\geq 1$

nếu $a=3b$

$\Leftrightarrow \sqrt{x^{2}+x+1}=3\sqrt{x-1}$

$\Leftrightarrow x^{2}+x+1=9x-9$

$x^{2}-8x+10=0$

$\Delta '=4^{2}-10=6$

phương trình có 2 nghiệm $\left\{\begin{matrix} x=4+\sqrt{6}\left ( TMDK \right ) & \\ x=4-\sqrt{6} & \left ( TMDK \right ) \end{matrix}\right.$

vậy phương trình có 2 nghiệm $x=\left \{ 4+\sqrt{6},4-\sqrt{6} \right \}$

 

 

  d =10

  S =17+10.3=47




#489712 Trận 6 - Phương trình nghiệm nguyên, đồng dư, chia hết

Đã gửi bởi nguyentrungphuc26041999 on 30-03-2014 - 20:24 trong Thi giải toán Marathon cấp THCS 2014

ta có ,với $x=1$

$\sqrt{2025x^{2}+2012x+3188}=\sqrt{2025+2012+3188}=85$

$\Rightarrow 85=2013-2011y+2094$

$\Leftrightarrow 4022=2011y$

$\Leftrightarrow y=2$ (thoả mãn $y$ là số tự nhiên)

Cặp số nguyên $\left ( x,y \right )=\left ( 1,2 \right )$ thoả mãn phương trình.

Vậy luôn tồn tại cặp số nguyên $\left ( x,y \right )$ thoả mãn phương trình

 

  d =9

 S =17+9x3 = 44




#487261 Trận 5 - toán rời rạc

Đã gửi bởi nguyentrungphuc26041999 on 16-03-2014 - 20:38 trong Thi giải toán Marathon cấp THCS 2014

Cho bàn cờ vua $8 \times 8$. Theo thứ tự từ trái qua phải, từ trên xuống dưới, ta làm việc sau:

Trong ô cờ thứ nhất đặt 1 hạt ngô

Trong ô cờ thứ hai đặt 2 hạt ngô

Trong ô cờ thứ ba đặt 4 hạt ngô

...

Trong ô cờ thứ 64 đặt $2^{63}$ hạt ngô.

 

Một con mã ô đầu tiên của bàn cờ, nó đi lòng vòng và ăn các hạt ngô trong ô nó nhảy đến( con mã di chuyển theo hình chữ L - 3 ô như đối với môn cờ vua) nhưng nó không ăn ở ô đầu tiên và không nhảy trở lại ô đầu tiên. Sau mỗi lần nó ăn người ta lại đặt số ngô bằng số ngô ban đầu vào trong ô đó. Sau khi con mã đi xong nó quay trở về ô đầu tiên và ăn nốt hạt ngô ở ô đó.

 

Hãy CM rằng số ngô mà con mã ăn chia hết cho 3.

 

Ta có 

$2^{2a}=4^{k}\equiv 1^{a}\left ( mod 3 \right )$

suy ra $2^{a}$ chia $3$ dư $1$

$2^{2b+1}=2.4^{k}\equiv 2.1^{b}\left ( mod3 \right )$

$2^{2b+1}$ chia $3$ dư $2$

Quay trở lại bài toán,đánh thứ tự cột từ trái sang phải là $a,b,c,d,e,f,g,h$,đánh thứ tự cột là $1,2,3,4,5,6,7,8$

gọi ô có số hạt ngô khi chia cho $3$ dư $1$ là ô trắng,gọi ô có số hạt ngô khi chia cho $3$ dư $2$ là ô đen

dễ thấy ô $a1$ là ô trắng,trong mỗi hàng, ô đen và ô trắng xen kẽ nhau theo nên ô cuối của hàng 1 (ô $a8$) là ô đen do mỗi hàng có 8 ô.

vì thế ô cuối cùng của hàng $2$ là ô trắng vì thế ô đầu tiên của hàng 2 là ô đen do mỗi hàng có 8 ô.

Lập luận như thế ta cũng có ở cột $a$ ô trắng và ô đen xen kẽ nhau.Nhưng do ô đen và ô trắng mỗi hàng xen kẽ nhau nên ô đen và ô trắng ở mỗi hàng cũng xen kẽ nhau.

Vậy ô trắng và ô đen đều xen kẽ nhau ở mỗi hàng và mỗi cột ( theo bàn cờ vua )

Dễ thấy sau mỗi lượt đi,con mã nhảy từ trắng sang ô đen và ô đen sang ô trắng.

Vì thế,sau mỗi 2 lần đi,con mã sẽ nhảy quá lanf ô trắng và 1 lần ô đen,số hạt ngô con ngựa ăn được sẽ chia hết cho 3(cho dù chúng có nhảy qua ô chúng đã đi qua hay không vì sau mỗi lần nó ăn,người ta lại đặt lại số ngô nó ăn)do $2^{2a}+2^{2b+1}\equiv 1+2\left ( mod 3 \right )$ hay chia hết cho $3$ 

Ta có, con ngựa xuất phát ở ô trắng và kết thúc ở ô trắng nên số nước đi của chúng là lẻ nhưng do ở ô đầu tiên nó không ăn hạt ngô đó,vì vậy số nước đi của nó là chẵn,nên số hạt ngô nó ăn chia hết cho 3 (đpcm)




#485216 Trận 4 - Bất đẳng thức

Đã gửi bởi nguyentrungphuc26041999 on 28-02-2014 - 22:16 trong Thi giải toán Marathon cấp THCS 2014

Ta có 

$x^{2}+y^{2}\geq 2xy$ (mọi $x,y\in R$)

$\Rightarrow \left ( x+y \right )^{2}\geq 4xy$

$\Rightarrow \left ( x+y \right )^{3}+\left ( x+y \right )^{2}\geq \left ( x+y \right )^{3}+4xy\geq 2$

$\Rightarrow \left ( x+y \right )^{3}+\left ( x+y \right )^{2}\geq 2$

$\Rightarrow \left ( x+y-1 \right )\left [ \left ( x+y \right )^{2}+2\left ( x+y \right )+2 \right ]\geq 0$

Do $\left ( x+y \right )^{2}+2\left ( x+y \right )+2= \left ( x+y+1 \right )^{2}+1>0$

$\Rightarrow x+y-1 \geq 0$

$\Rightarrow x+y \geq 1$

Theo bất đăng thức Bunnhiacopxki 

$2\left ( x^{2}+y^{2} \right )\geq \left ( x+y \right )^{2}\geq 1$

$x^{2}+y^{2}\geq \frac{1}{2}$

ta lại có 

$P=3\left ( x^{4}+y^{4}+x^{2}y^{2} \right )-2\left ( x^{2}+y^{2} \right )+1$

$\Rightarrow P=3\left [ \frac{2\left ( x^{4}+y^{4} \right )}{4}+\frac{x^{4}+y^{4}+2x^{2}y^{2}}{2} \right ]-2\left ( x^{2}+y^{2} \right )+1$

Theo bất đẳng thức bunhiacopxki ta có

$P\geq 3\left [ \frac{\left ( x^{2}+y^{2} \right )^{2}}{4}+\frac{\left ( x^{2}+y^{2} \right )^{2}}{2} \right ]-2\left ( x^{2}+y^{2} \right )+1$

$\Rightarrow P\geq \frac{9}{4}\left ( x^{2}+y^{2} \right )^{2}-2\left ( x^{2}+y^{2} \right )+1$

đặt $x^{2}+y^{2}=a\left ( a\geq \frac{1}{2} \right )$

ta sẽ tìm GTNN của $\frac{9}{4}a^{2}-2a+1$

ta có $\frac{9}{4}a^{2}-2a+1=2\left ( a^{2}-a+\frac{1}{4} \right )+\frac{a^{2}+2}{4}$

$=2\left ( a-\frac{1}{2} \right )^{2}+\frac{\left ( \frac{1}{2} \right )^{2}+2}{4}\geq \frac{9}{16}$

$\Rightarrow P\geq \frac{9}{16}$ 

Dấu bằng xảy ra khi $x=y=\frac{1}{2}$

 

Điểm 10.




#483213 Trận 3 - Hình học

Đã gửi bởi nguyentrungphuc26041999 on 15-02-2014 - 12:57 trong Thi giải toán Marathon cấp THCS 2014

tuy không được tham gia trận 3 nhưng em cũng xin đóng góp cách làm của mình

$AM$ cắt cạnh $BE$ và $BC$ lần lượt tại $J$ và $K$ 

$BN$ cắt $AC$ tại $L$ ,$CP$ cắt $AB$ tại $T$

áp dụng định lý Mê- nê-la uýt liên tiếp, ta có 

xét $\bigtriangleup BEF$ có $A\in BF,M\in EF,J\in BE$

$\Rightarrow \frac{ME}{MF}.\frac{AF}{AB}.\frac{JB}{JE}=1\left ( 1 \right )$

xét $\bigtriangleup BEC$ có $A\in EC,J\in BE ,K\in BC$

$\Rightarrow \frac{JE}{JB}.\frac{KB}{KC}.\frac{AC}{AE}\left ( 2 \right )$

nhân vế theo vế của $\left ( 1 \right )$ và $\left ( 2 \right )$ ta có 

$\Rightarrow \frac{ME}{MF}.\frac{AF}{AB}.\frac{KB}{KC}.\frac{AC}{AE}=1$

tương tự ta cũng có 

$\frac{PD}{PE}.\frac{TA}{TB}.\frac{BC}{CD}.\frac{CE}{CA}=1$

$\frac{NF}{ND}.\frac{LC}{LA}.\frac{AB}{BF}.\frac{BD}{CB}=1$

nhân 3 hệ thức trên lại với nhau ta được 

$\frac{ME}{MF}.\frac{PD}{PE}.\frac{NF}{ND}.\frac{KB}{KC}.\frac{TA}{TB}.\frac{LC}{LA}.\frac{FA}{FB}.\frac{CE}{AE}.\frac{DB}{DC}=1\left ( * \right )$

áp dụng định lý Cê-va cho $\bigtriangleup ABC$ có $AD,BE,CF$ đồng quy và $\bigtriangleup DEF$ có $DM,FP,EN$ đồng quy,

$\frac{FA}{FB}.\frac{CE}{EA}.\frac{DB}{DC}=1$ và $\frac{ME}{MF}.\frac{PD}{PE}.\frac{NF}{ND}=1\left ( ** \right )$

từ $\left ( * \right )$ và $\left ( ** \right )$ suy ra $\frac{KB}{KC}.\frac{TA}{TB}.\frac{LC}{LC}=1$

theo định lý Cê-va đảo ta có $AM,BN,CP$ đồng quy (đpcm)

untitled.JPG  




#479104 Trận 2 - PT, HPT

Đã gửi bởi nguyentrungphuc26041999 on 26-01-2014 - 09:38 trong Thi giải toán Marathon cấp THCS 2014

hệ tương đương với $\left\{\begin{matrix} 2x^{2}-5xy+3y^{2}=0 & \\ 4x^{2}-6x+1=y^{2}-3y & \end{matrix}\right.$

$\Leftrightarrow \left\{\begin{matrix} \left ( x-y \right )\left ( 2x-3y \right )=0 & \\ 4x^{2}-6x+1=y^{2}-3y & \end{matrix}\right.$

trường hợp 1 nếu $x=y$

hệ trở thành $\left\{\begin{matrix} x=y & \\ 3x^{2}-3x +1=0& \end{matrix}\right.$

$\Delta =-3< 0$ nên hệ vô nghiệm

trường hợp 2 

nếu $2x=3y$

hệ trở thành $\left\{\begin{matrix} 2x=3y & \\ 8y^{2}-6y+1=0 & \end{matrix}\right.$

$\left\{\begin{matrix} 2x=3y & \\ \left ( 4y-1 \right )\left ( 2y-1 \right )=0 & \end{matrix}\right.$

nếu $y=\frac{1}{4}$ $\Rightarrow x=\frac{3}{8}$

 nếu $y=\frac{1}{2}$ $\Rightarrow x=\frac{3}{4}$

Vậy $\left ( x,y \right )=\left \{ \left ( \frac{3}{8},\frac{1}{4} \right ),\left ( \frac{3}{4},\frac{1}{2} \right ) \right \}$

 

d = 10

S = 36




#481644 Trận 2 - PT, HPT

Đã gửi bởi nguyentrungphuc26041999 on 07-02-2014 - 17:11 trong Thi giải toán Marathon cấp THCS 2014

Hai toán thủ nguyentrungphuc26041999  và  Frankie nole có cùng địa chỉ IP và có lời giải giống hệt nhau. Điều này là gian lận.

 

BTC đã chấm điểm cho nguyentrungphuc26041999 và cảnh cáo toán thủ này. 

 

Toán thủ Frankie nole bị loại vì hành vi gian lận

xin lỗi BTC nhưng hôm đó,mạng nhà em bị rớt,mất mạng vài tuần nên đến nhà toán thủ frankynole để làm,do bài này khá dễ nên làm giống nhau là chuyện bình thường vì đây là cách mà phần lớn các toán thủ đều làm.




#475771 Trận 1 - Phương trình nghiệm nguyên ...

Đã gửi bởi nguyentrungphuc26041999 on 06-01-2014 - 19:57 trong Thi giải toán Marathon cấp THCS 2014

chết thật,sót mất $m=0$ loại

ghi nhầm $n$ chẵn lẻ nữa




#475131 Trận 1 - Phương trình nghiệm nguyên ...

Đã gửi bởi nguyentrungphuc26041999 on 03-01-2014 - 23:00 trong Thi giải toán Marathon cấp THCS 2014

Giả sử $m\neq 1$

$\Rightarrow m> 1$

Khi $n$ lẻ $\Rightarrow n^{2}+1\vdots 2$

$\Rightarrow \left ( n^{2}+1 \right )^{2^{k}}\equiv 1\left ( mod4 \right )$

$\Rightarrow \left ( 44n^{3}+11n^{2}+10n+2 \right )\equiv 2\left ( mod4 \right )$

$\Rightarrow VT\equiv 2\left ( mod4 \right )$

$\Rightarrow N\vdots 2$

$\Rightarrow N^{m}\vdots 4$

Suy ra vô lý 

khi n chẵn,$44n^{3}+11n^{2}+10n+2\vdots 2$

$N\vdots 2$

đặt $N=2^{a}b$ với $b\equiv 1\left ( mod2 \right )$ 

đặt $n^{2}+1=2c$ với $c\equiv 1\left ( mod2 \right )$

Phương trình trở thành 

$\left ( 2c \right )^{2^{k}}.\left ( 44n^{3}+11n^{2}+10n+2 \right )=\left ( 2^{a}b \right )^{m}$

$2^{2^{k}}c^{2^{k}}.\left ( 44n^{3}+11n^{2}+10n+2 \right )= 2^{am}b^{m}$

do $b$ lẻ  nên $2^{k}=am$

$\Rightarrow m$ chẵn

$\left ( 44n^{3}+11n^{2}+10n+2 \right )$  là số chính phương 

$44n^{3}+11n^{2}+10n+2=44n^{3}+12n^{2}+8n-\left ( n^{2}-2n-2 \right )\equiv -n^{2}+2n+2\left ( mod4 \right )$

$\Rightarrow -n^{2}+2n+2=3-\left ( n-1 \right )^{2}$

nếu $\left ( n-1 \right )^{2}\equiv 1\left ( mod 4 \right )$

$\Rightarrow 3-\left ( n-1 \right )^{2}\equiv 2\left ( mod 4 \right )$

suy ra $44n^{3}+11n^{2}+10n+2$ không phải số chính phương 

nếu $\left ( n-1 \right )^{2}\equiv 0\left ( mod4 \right )$

$\Rightarrow 3-\left ( n-1 \right )^{2}\equiv 3\left ( mod4 \right )$

suy ra $44n^{3}+11n^{2}+10n+2$ không là số chính phương 

Suy ra $m=1$

 

Điểm bài :6đ ( chổ màu đỏ xem lại cách lập luận)

Nhầm lẫn nghiêm trọng giữa hai trường hợp $n$ lẻ và $n$ chẵn.

S = 16,3 + 6*3 = 34.3




#444127 toán tổng hợp giới hạn dãy số, bđt, hình phẳng, pt hàm

Đã gửi bởi nguyentrungphuc26041999 on 19-08-2013 - 20:14 trong Hàm số - Đạo hàm

hôm này trường mình thi đề này cx vừa sức. ae vào chém cùng nào.

 

câu 1. giải pt

 

$x^2+2x\sqrt{x-\dfrac{1}{x}}=3x+1$

 

câu 2. cho 2 số thực dương $x, y  \geq 1$ và 3(x+y)=4xy

 

tìm min, max của $P=x^3+y^3+3(\dfrac{1}{x^2}+\dfrac{1}{y^2})$

 

câu 3. giải hpt

 

$x=3y^3+2y^2$

$y=3z^3+2z^2$

$z=3x^3+2x^2$

 

câu 4. có bao nhiêu stn có 7 chữ số khác nhau thoả mãn 2 đk sau

 

+Có 3 chữ số 3,4,5 đứng liền nhau.

+Có 2 chữ số 7,9 đứng liền nhau

 

câu 5. Cho ▲ABC có trực tâm H. Đường phân giác ngoài của $\widehat{BHC}$ cắt AB, AC tại D và E. Đường p/g trong của $\widehat{BAC}$ cắt đường tròn ngoại tiếp tam giác ADE tại K.

 

CMR HK đi qua trung điểm BC.

 

Câu 6. Cho {Un} xác định như sau

 

$U_1 = 3$; $U_{n+1} = \dfrac{U_n^2-2}{2U_n-3}$ với $n \in N^*$

 

xác định công thức tổng quát của $U_n$

 

câu 7. f(x) xác định và liên tục trên R thoả mãn $f(f(x))=3f(x)-2x \forall x \in R$ và f(1)=1

 

tìm tất cả các  hàm như thế

câu 2

theo giả thiết ta có

$\frac{1}{x}+\frac{1}{y}= \frac{4}{3}$

ta có

$\left ( x^{3}+y^{3} \right )\left ( \frac{1}{x}+\frac{1}{y} \right )\geq \left ( x+y \right )^{2}$

suy ra $\left ( x^{3}+y^{3} \right )\geq \frac{3\left ( x+y \right )^{2}}{4}$

do$\left ( x+y \right )\left ( \frac{1}{x}+\frac{1}{y} \right )\geq 4$

nên$x+y\geq 3$

suy ra$\frac{3}{4}\left ( x^{3}+y^{3} \right )\geq \left ( x+y \right )^{2}\geq 9$

hay$x^{3}+y^{3}\geq 12$

$P\geq 12+3\left ( \frac{1}{x^{2}}+\frac{1}{y^{2}} \right )\geq 12+\frac{3}{2}\left ( \frac{1}{x}+\frac{1}{y} \right )^{2}=32$

suy ra MIN P= 32 

dấu bằng tự tìm




#443190 Toán tuổi thơ-Toán tuôi trẻ

Đã gửi bởi nguyentrungphuc26041999 on 15-08-2013 - 22:32 trong Góp ý cho diễn đàn

Em mạnh dạn nêu ý kiến. Tại sao các mod ko lập 1 topic tổng hợp các bài toán tuôi thơ - tuổi trẻ số ms nhất ạ. Các mod chj cần cung cấp đề cho các thành viên chưa có báo.

p/s: Các mod khóa topic ấy vào ko cho các thành viên đăng bài chỉ có thể xem bài ạ.  :icon6:  :lol:  :lol:

Nếu có gì ko phải mong BQT tha lỗi cho em  :mellow:  :mellow:

nếu thế có người giải ,đăng lên thì nhiều khả năng sẽ có nhiều bài gửi về TTT hay THvsTT hơn,

như thế không hay đâu.

ít nhất em xin góp ý mở topic này để thông tin nhanh hơn.




#446386 Toán logic

Đã gửi bởi nguyentrungphuc26041999 on 30-08-2013 - 21:19 trong IQ và Toán thông minh

Mấy bài này khó quá

trả lời mấy câu trắc nghiệm

1.A; 2.A;3.B;4.E;5.B;6.G;7.D;8.B;9.G;10.D;12.C;13.G;14.H;15.C;16.C;17.2B;18.D;19.D;20.B




#477541 toán bất đẳng thức

Đã gửi bởi nguyentrungphuc26041999 on 16-01-2014 - 16:17 trong Bất đẳng thức và cực trị

mấy bác giải giùm em với !!!!

 

1/Cho a,b,c >0 và a+b+c=6abc. chứng minh rằng:

$\frac{bc}{a^3(c+2b)}+\frac{ac}{b^3(a+2c)}+\frac{ab}{c^3(b+2a)}$$\geq 2$

 

 

2/Cho $\alpha ,\beta ,\gamma \geq 0$. tìm min:

M= $\frac{\alpha x}{y+z}+\frac{\beta y}{x+z}+\frac{\gamma z}{y+x}$

 

(x,y,z>0)

 

thank các bác. em làm lính mới chưa bít cách đăng bài 

1 đặt $\left\{\begin{matrix} \frac{1}{a}=x & \\ \frac{1}{b}=y & \\ \frac{1}{c}=z & \end{matrix}\right.$

khi đó ta có 

$xy+yz+zx=6$

bất đẳng thức tương đương$\sum \frac{x^{3}}{y+z}\geq 2$

theo bất đẳng thức BCS 

$\sum \frac{x^{3}}{y+z}=\sum \frac{x^{4}}{2xy+xz}\geq \frac{\left ( x^{2}+y^{2}+z^{2} \right )^{2}}{3\left ( xy+yz+zx \right )}\geq \frac{\left ( xy+yz+zx \right )^{2}}{3\left ( xy+yz+zx \right )}=2$

dấu bằng tự tìm




#498353 Topic tổng hợp các bài toán về phương trình nghiệm nguyên.

Đã gửi bởi nguyentrungphuc26041999 on 11-05-2014 - 10:07 trong Số học

220/ 

a. $\sqrt{x}+\sqrt{x+3}=y$

b.Tìm nghiệm nguyên của hệ phương trình

$\left\{\begin{matrix} 2x+3y=8 & & \\ 5y+3z=1 & & \end{matrix}\right.$

c.$.x+y+z=xyz$

d.$\sqrt{x}+\sqrt{y}=\sqrt{1998}$

e.$3x^2+5y^2=12$

f.$3(x^2+y^2+xy)=x+8y$

g.$x(x+1)(x+2)(x+3)=y^2$

h.$\frac{1}{x^2}+\frac{1}{y^2}+\frac{1}{z^2}+\frac{1}{t^2}=1$

g,$\Leftrightarrow \left ( x^{2}+3x+2 \right )\left ( x^{2}+3x \right )=y^{2}$

$\left ( x^{2}+3x+1-y \right )\left ( x^{2}+3x+1+y \right )=1$




#498359 Topic tổng hợp các bài toán về phương trình nghiệm nguyên.

Đã gửi bởi nguyentrungphuc26041999 on 11-05-2014 - 10:24 trong Số học

220/ 

a. $\sqrt{x}+\sqrt{x+3}=y$

b.Tìm nghiệm nguyên của hệ phương trình

$\left\{\begin{matrix} 2x+3y=8 & & \\ 5y+3z=1 & & \end{matrix}\right.$

c.$.x+y+z=xyz$

d.$\sqrt{x}+\sqrt{y}=\sqrt{1998}$

e.$3x^2+5y^2=12$

f.$3(x^2+y^2+xy)=x+8y$

g.$x(x+1)(x+2)(x+3)=y^2$

h.$\frac{1}{x^2}+\frac{1}{y^2}+\frac{1}{z^2}+\frac{1}{t^2}=1$

c,giả sử $x\geq y\geq z$

với $x=y=z=0$ đúng

ta có $1=\frac{1}{xy}+\frac{1}{yz}+\frac{1}{xz}\leq \frac{3}{z^{2}}$

$\Rightarrow z=1$

$\Leftrightarrow \left ( x-1 \right )\left ( y-1 \right )=2$




#498355 Topic tổng hợp các bài toán về phương trình nghiệm nguyên.

Đã gửi bởi nguyentrungphuc26041999 on 11-05-2014 - 10:14 trong Số học

220/ 

a. $\sqrt{x}+\sqrt{x+3}=y$

b.Tìm nghiệm nguyên của hệ phương trình

$\left\{\begin{matrix} 2x+3y=8 & & \\ 5y+3z=1 & & \end{matrix}\right.$

c.$.x+y+z=xyz$

d.$\sqrt{x}+\sqrt{y}=\sqrt{1998}$

e.$3x^2+5y^2=12$

f.$3(x^2+y^2+xy)=x+8y$

g.$x(x+1)(x+2)(x+3)=y^2$

h.$\frac{1}{x^2}+\frac{1}{y^2}+\frac{1}{z^2}+\frac{1}{t^2}=1$

a,$\Leftrightarrow 2x+3+2\sqrt{x\left ( x+3 \right )}=y^{2}$

nếu $x=0$  không thoả mãn 

nếu $x=3$ không thoả mãn

nếu $x\left ( x+3 \right )=k^{2}$

$\Leftrightarrow \left ( 2x+3-k \right )\left ( 2x+3+k \right )=9$




#444005 Topic nhận đề bất đẳng thức, cực trị hoặc toán rời rạc

Đã gửi bởi nguyentrungphuc26041999 on 19-08-2013 - 13:03 trong Bài thi đang diễn ra

Họ và tên : Nguyễn Trung Phúc.

Đang học lớp 9A , Trường THCS Đặng Thai Mai, thành phố Vinh ,Tỉnh Nghệ An

Đề

Cho $a,b,c$ là các số thực dương, thoả mãn $a+b+c=3$

Chứng minh rằng

$\sum \frac{1}{4a^{2}+b^{2}+c^{2}}\leq \frac{1}{2}$

Lời giải

Sử dụng bất đẳng thức Bunhiacopxki dạng phân thức ta có

$\frac{9}{4a^{2}+b^{2}+c^{2}}= \frac{\left ( a+b+c \right )^{2}}{2a^{2}+\left ( a^{2}+b^{2} \right )+\left ( a^{2}+c^{2} \right )}\leq \frac{a^{2}}{2a^{2}}+\frac{b^{2}}{a^{2}+b^{2}}+\frac{c^{2}}{a^{2}+c^{2}}$

từ đây suy ra

$\sum \frac{9}{4a^{2}+b^{2}+c^{2}}\leq \frac{3}{2}+\sum \left ( \frac{b^{2}}{a^{2}+c^{2}} \right )=\frac{3}{2}+3= \frac{9}{2}$(đpcm)

dấu bằng xảy ra khi $a=b=c=1$




#499766 TOPIC các bài đất đẳng thức THCS

Đã gửi bởi nguyentrungphuc26041999 on 18-05-2014 - 09:57 trong Bất đẳng thức và cực trị

Bài tiếp :

5) Cho $a,b,c$ là độ dài cạnh của 1 tam giác.

Chứng minh rằng : $\frac{a}{b+c-a}+\frac{b}{a+c-b}+\frac{c}{a+b-c}\geq 3$ (Giải bằng 2 cách)  ~O)

$\sum \frac{a}{b+c-a}\geq 3\sqrt[3]{\frac{abc}{\prod \left ( a+b-c \right )}}\geq 3$

hoặc dùng Svacs




#448292 tim min va max của F=xy +yz+zx-kxyz

Đã gửi bởi nguyentrungphuc26041999 on 06-09-2013 - 20:56 trong Bất đẳng thức và cực trị

cho k là số thực co dinh va x,y,z thỏa mãn x+y+z=1

tim min va max của

F=xy +yz+zx-kxyz

ta có 

$xy+yz+zx\leq \frac{\left ( x+y+z \right )^{2}}{3}=\frac{1}{3}$

$kxyz\leq k\frac{\left ( x+y+z \right )^{3}}{27}= \frac{k}{27}$

$\Rightarrow xy+yz+zx+kxyz\leq \frac{1}{3}+\frac{k}{27}$




#445750 Thêm chương trình

Đã gửi bởi nguyentrungphuc26041999 on 27-08-2013 - 18:06 trong Góp ý cho diễn đàn

Sao diễn đàn ta không thêm chương trình TIỂU HỌC nhỉ??? Mình có thằng em đang học lớp 5 cần học toán trên mạng nè. Diễn đàn chúng ta thừa thông minh để hướng dẫn các em chứ nhỉ. Nếu thêm tiểu học chắc lượng truy cập tăng đáng kể đấy. Lại có đất cho các em nữa  :wub:  :wub: . Mình nghĩ như vậy.

mấy cái đó không cần thiết cho lắm.nếu có chương trình cho tiểu học thì bao lâu sau mới có được hưởng ứng của học sinh tiểu học.Với lại việc đó chưa thực sự cần thiết vì lượng kiến thức ở tiểu học còn ít chưa chuyên sâu. 




#445470 THCS

Đã gửi bởi nguyentrungphuc26041999 on 25-08-2013 - 22:35 trong Số học

Giúp em xong trước trưa 26/8/2013

Chứng minh rằng: với n>1 thì 2n -1 không phải là số chính phương

do $n> 1$ 

$\Rightarrow 2^{n}\vdots 4$

$2^{n}-1\equiv 3\left ( mod4 \right )$

mà $k^{2}\equiv 1\left ( mod4 \right )$hoặc$k^{2}\equiv 0\left ( mod4 \right )$

suy ra $2^{n}-1$ không thể là số chính phương